LSAT and Law School Admissions Forum

Get expert LSAT preparation and law school admissions advice from PowerScore Test Preparation.

User avatar
 Dave Killoran
PowerScore Staff
  • PowerScore Staff
  • Posts: 5850
  • Joined: Mar 25, 2011
|
#92212
Complete Question Explanation
(The complete setup for this game can be found here: lsat/viewtopic.php?f=370&p=92208)

The correct answer choice is (B).

The condition in the question stem creates the following sequence:

G1-Q3-d1.png

Because this block includes R and T (the two variables from the third rule), the block in this sequence can only go in two positions: 1-4 or 3-6.

If the block is in 1-4, then Q, W, and P are in the final three spaces, with the rule that Q > W.
If the block is in 3-6, then P must be last from the second rule, and Q is 1st and W is 2nd from the first rule. These are the two options:

G1-Q3-d2.png

Accordingly, answer choice (B) is correct.
User avatar
 KitKat
  • Posts: 3
  • Joined: Jul 15, 2022
|
#96199
Hi,

I did get the two templates indicated when working through the game on my own with the sequence in block 1-4 and block 3-6, but I wavered between A & B as in both set ups Q-P and S-P . I ultimately chose A but am not clear on why S-P is the MBT when Q-P is also present in both scenarios.

Thank you!
 Adam Tyson
PowerScore Staff
  • PowerScore Staff
  • Posts: 5153
  • Joined: Apr 14, 2011
|
#96218
Look closely again at that first template, KitKat, where T is first. The last three spaces show (Q-W, P), and that means that within those three spaces there will be a Q somewhere before a W, and there will also be a P somewhere. But the P is not connected in any way to the Q or the W; it's a free agent within those three spaces. That allows for three possible orders of those variables:

QWP
QPW
PQW

Note that one of those possible orders has the P before the Q, proving that answer A does not have to be true.

Get the most out of your LSAT Prep Plus subscription.

Analyze and track your performance with our Testing and Analytics Package.